User avatar
 
ohthatpatrick
Thanks Received: 3808
Atticus Finch
Atticus Finch
 
Posts: 4661
Joined: April 01st, 2011
 
This post thanked 3 times.
 
 

Re: Q14 - Newspaper subscriber: Arnot's editorial argues

by ohthatpatrick Fri Dec 31, 1999 8:00 pm

Question Type:
Flaw (which expresses a flaw)

Stimulus Breakdown:
Conclusion: (this conclusion is false) If we made certain fundamental changes in govt, we would NOT virtually eliminate our most vexing social ills.
Evidence: Arnot makes an argument that it WOULD virtually eliminate our most vexing social ills, but his argument depends on a dubious assumption (that we can trust the govt to act in the public's interest)

Answer Anticipation:
This is one of the ten classic flaws: Unproven vs. Untrue.

If someone made an argument, but that argument relied on a dubious assumption, then we can say "You failed to prove your conclusion." We CANNOT say, "Your conclusion is false". The more typical version of this flaw feels like, "Scientists have never been able to prove that X is true. Thus, X is not true."

Correct Answer:
A

Answer Choice Analysis:
(A) Does this match the Core? Yes. The author's conclusion IS inded repudiating a claim ("clearly this conclusion is false") and the premise is simply that Arnot's argument depends on a dubious assumption (it's an inadequate argument).

(B) Does this match the Core? No. There's nothing in the argument that we can match up with "a change that is REQUIRED for eliminating social ills". Furthermore, this answer choice describes another of the ten classic flaws: Conditional Logic Flaw (i.e. "Nec vs. Suff" ... required vs. guarantee) There is no conditional logic in this argument.

(C) Would this weaken? Almost but no. It would weaken if it said "even if some assumptions used to justify a conclusion are false, that conclusion might nonetheless be true". This answer choice says that backwards.

(D) Does this match the Core? No. This is called a "Straw Man", for what it's worth. If Arnot had said "we should raise the minimum wage" and then our author had attacked the idea that "entry level jobs should pay just as well as highly skilled jobs", that would be what (D) describes.

(E) Does this match the Core? No. This is another classic flaw, called Equivocation (but basically just remember it as the flaw where they say a certain "word" was used in two different ways). Government is being used consistently throughout the argument.

Takeaway/Pattern: This is the more subtle form of the Unproven vs. Untrue flaw. Because someone believed claim X for a sketchy reason, the author concludes that claim X is false. Claim X might still be true. All we can reasonably conclude is that "we have not yet been convinced that claim X is true."

#officialexplanation
 
kimraan87
Thanks Received: 1
Forum Guests
 
Posts: 2
Joined: September 08th, 2011
 
This post thanked 1 time.
 
 

Q14 - Newspaper subscriber: Arnot's editorial argues

by kimraan87 Mon Nov 07, 2011 12:50 am

I am finding it difficult to understand the flaw in this question and just don't see how A is the answer. Can someone please help explain why the other answers are wrong?
User avatar
 
ohthatpatrick
Thanks Received: 3808
Atticus Finch
Atticus Finch
 
Posts: 4661
Joined: April 01st, 2011
 
This post thanked 21 times.
 
 

Re: Q14 - Newspaper subscriber: Arnot's editorial argues

by ohthatpatrick Wed Nov 09, 2011 5:27 pm

Hey, hey.

This flaw is what we call "Absence of Evidence" -- essentially, it assumes that someone's CONCLUSION is wrong just because his/her EVIDENCE is sketchy or nonexistent.

There are two common manifestations:
1. No evidence to the contrary means something is true.
Many people worry that cell phones cause cancer. However, no one has yet been able to prove that cell phones do cause cancer. Hence, cell phones are safe.


This form takes a certain idea, we'll call it 'A' (cell phones cause cancer), and reasons that since no one has proven that A is true, we may conclude that A is false.

In reality, a reasonable person would believe we still don't know whether A is true or false. This author certainly hasn't given us any evidence of her own to justify that A is false.

You can try to prove God exists by challenging your opponent to "prove He doesn't".

Or you could just as easily try to prove God does NOT exist by challenging your opponent to "prove He does".

Both of those are hollow arguments. The failure to refute a certain idea does not prove its truth. The failure to prove an idea does not refute it.

In its most nauseating wording, the test has phrased this flaw as "the author confuses absence of evidence with evidence of absence".

2. Sketchy evidence for a claim means that the claim itself is false.
A recent study claimed that Mars once had oceans. However, the researchers conducting the study had their methodology torn to shreds during peer review. Furthermore, the head of the study has previously published many false so-called "findings". Thus, Mars has never had oceans.

This one is more subtle. By undermining the trustworthiness of the evidence, we should be more dubious of a certain conclusion or think that a certain claim is baseless. But that's different from concluding the conclusion/claim is actually wrong! Even if we presently don't trust this sketchy research team's findings, we can't assume the findings themselves are wrong.

I could tell you that I asked a magic 8-ball whether the molecular composition of water is H20, and it said "yes". You could say, "magic 8-balls are a completely dubious source of information about molecular composition". But would you want to further conclude, "thus, water is NOT H20."?

You can believe a true claim for a crappy reason. Pointing out that someone's reason for believing a claim is crappy doesn't prove that the claim itself is wrong.

So that's the flaw in #14. Pointing out that Arnot's evidence is dubious does not prove that his conclusion is false. It may well be a true conclusion, even if Arnot believes it for dubious reasons.

(B) describes Ye Olde flaw of Necessary/Sufficient conflation. in friendlier terms, Nec/Suff conflation means that there was conditional logic in the premise, but the author interpreted it incorrectly in the conclusion.

For example: Any time you give a woman flowers, she smiles. Since I just saw Molly smiling, someone must have given her flowers.

(C) is the backwards re-mix of (A). What we WANT to hear is "it fails to consider that, even if an argument's PREMISE is false, the CONCLUSION may nonetheless be true"

(D) we don't have any means to support the idea that the author has distorted Arnot's argument. And this answer won't address the real problem that pointing out a shaky assumption underlying someone's argument does NOT prove that the conclusion of that argument was false.

(E) this refers to equivocation, or shifting meanings. This answer is typically wrong when you see it. In order for it to legitimately be the flaw, you need to see two different uses of the same word.

Example: Brad's parents complain that Brad is not a responsible young adult. But clearly they're mistaken. After all, Brad freely admits that he was responsible for setting fire to the back porch.

Hope this helps.
 
kimraan87
Thanks Received: 1
Forum Guests
 
Posts: 2
Joined: September 08th, 2011
 
 
 

Re: Q 14 - Arnot's editorial argues...

by kimraan87 Sun Nov 13, 2011 1:38 pm

That definitely helps clear it up, thank you!
 
irenaj
Thanks Received: 0
Forum Guests
 
Posts: 18
Joined: August 31st, 2011
 
 
 

Re: Q 14 - Arnot's editorial argues...

by irenaj Fri Nov 18, 2011 11:59 pm

Hello ohthatpatrick, this question reminds me of another flaw of reasoning question in recent years (about whether a subject concerning math should be necessarily taught by math department), and the correct answer is also like: one inadequate reasoning opposing the proposal does not demonstrate the proposal is correct.

I choose A by POE. But I find valid conditional reasoning in this question and it really confuses me.

premise:
1. fundamental change--> eliminate
2. (fundamental change--> eliminate)--> public interest
3. public interest is a dubious assumption, therefore, ~public interest
concludion:
~public interest --> ( fundamental change-|-> eliminate)

I find it airtight and a typical standard conditional reasoning of A--> B, ~B--> ~A, which we confront million times in LR.

Then how come this as a flaw in the reasoning in this question?

Is it because "dubious assumption of public interest" does not equals to "~public interest"--that one finds it dubious does not mean it is actually dubious or actually not exist?

Could you point out my misunderstanding above please? Thanks in advance!
 
vik
Thanks Received: 8
Jackie Chiles
Jackie Chiles
 
Posts: 42
Joined: March 29th, 2011
 
This post thanked 2 times.
 
 

Re: Q14 - Newspaper subscriber: Arnot's editorial argues

by vik Sun Jan 15, 2012 7:17 pm

The news subscriber (NS) says Arnot's assumption is dubious. Hence, if the NS then said Arnot's conclusion is dubious, the NS would be correct. However, the NS says Arnot's conclusion is clearly false. This is a leap made by the NS (from dubious to clearly false). Thus the NS is flawed.
 
lhermary
Thanks Received: 10
Atticus Finch
Atticus Finch
 
Posts: 160
Joined: April 09th, 2011
 
 
 

Re: Q14 - Newspaper subscriber: Arnot's editorial argues

by lhermary Mon Apr 23, 2012 3:41 pm

I still don't get how A is right...

He/she is attacking the argument by saying the government can't be trusted (he's attacking the government's intentions). He's not saying that there isn't enough information given, therefore the argument is wrong....

help
User avatar
 
ohthatpatrick
Thanks Received: 3808
Atticus Finch
Atticus Finch
 
Posts: 4661
Joined: April 01st, 2011
 
This post thanked 3 times.
 
 

Re: Q14 - Newspaper subscriber: Arnot's editorial argues

by ohthatpatrick Tue Apr 24, 2012 4:33 pm

Good question. You used the phrase "there isn't enough information given" as a paraphrase for "an inadequate argument" from choice (A).

You're right. The Newspaper Subscriber didn't say "there isn't enough info". But he/she did say that the argument depended on a dubious assumption.

Would you accept "the argument made for a claim depends on a dubious assumption" is a paraphrase for "an inadequate argument has been given for a claim"?

If so, then (A) accurately describes the argument.

====
A few posts back, someone was attempting to convert this into conditional logic. This is a dangerous tendency many students develop once they learn about conditional logic. Please realize that only a minority of LR questions use conditional logic. It should NOT be our default approach. To briefly address the previous poster's notation, the conclusion here would be "fundamental changes --/--> eliminating social ills". There is nothing airtight about NS's argument, since his/her only premise, the last sentence, does not address "fundamental changes" or "eliminating social ills".
=====

Let me give one more shot at explaining (A).

The second to last poster put it marvelously simply: the extreme wording of the conclusion is our main gripe.

This Newspaper Subscriber would have a reasonable argument if he/she concluded "But clearly this conclusion is dubious/questionable/debateable".

The flaw that (A) describes relates to the fact that the author is saying that Arnot's conclusion is FALSE.

In order to prove that 'idea X' is FALSE, you have to really remove all possible doubts from my mind that 'idea X' could be true.

Has the NS done so with his/her premise?

Do you feel like the last sentence of this argument proves to you beyond the shadow of a doubt that Arnot's conclusion is FALSE?

I certainly don't.

I agree with you that the NS is attacking Arnot's argument by calling into question one of Arnot's assumptions. But when we "call into question" an argument, we're WEAKENING it, not REPUDIATING it.

The issue of whether or not the government can be trusted is related to, but distinct from, the issue of whether or not making certain fundamental changes in govt. would eliminate most social ills.

So, in and of itself, even if you told me that "the government CANNOT be trusted to act in the interest of the public", would that contradict the claim that "by making certain fundamental changes in government we would virtually eliminate our most vexing social ills"?

Those don't contradict each other.

So addressing Arnot's dubious assumption does not FALSIFY his conclusion; it merely weakens it.

Let me give a simpler analogy:

Arnot's editorial:
I flipped a coin to see which object is hotter, the Earth or the Sun. The Earth was 'heads', while the Sun was 'tails'. When I flipped the coin, it landed on tails. Therefore, the Sun is hotter than the Earth.

Newspaper subscriber:
Arnot's editorial argues that the Sun is hotter than the Earth. But clearly this conclusion is false. After all, the argument Arnot makes for this claim depends on the dubious assumption that a coin flip can be trusted to determine which of two bodies is hotter.

Here, the NS has just concluded "the Sun is NOT hotter than the Earth". Why? Because Arnot's argument that "the Sun IS hotter than the Earth" depended on a dubious assumption.

This is the essence of the Absence of Evidence flaw we were describing. Naturally, LSAT did a much better job of disguising it in Q14.

The moral to the story is: the truth/falsity of a conclusion is a separate issue from how reasonable/trustworthy someone's argument for that conclusion may be.
 
lhermary
Thanks Received: 10
Atticus Finch
Atticus Finch
 
Posts: 160
Joined: April 09th, 2011
 
 
 

Re: Q14 - Newspaper subscriber: Arnot's editorial argues

by lhermary Mon May 28, 2012 7:25 pm

That was incredibly helpful. Thanks
 
xinglipku
Thanks Received: 0
Forum Guests
 
Posts: 31
Joined: July 08th, 2012
 
 
 

Re: Q14 - Newspaper subscriber: Arnot's editorial argues

by xinglipku Thu Aug 30, 2012 4:28 am

Hi Patrick,

May I ask if Arnot's argument does assume what the newspaper subscriber says it (dubiously) assumes?

Thanks!
 
nlschultheis
Thanks Received: 0
Vinny Gambini
Vinny Gambini
 
Posts: 4
Joined: October 02nd, 2012
 
 
 

Re: Q14 - Newspaper subscriber: Arnot's editorial argues

by nlschultheis Tue Nov 27, 2012 6:33 pm

I was between A and D with this question and eventually chose D.

What mixed me up was that I noticed the distinction between the premise and the conclusion but I thought that the premise of "not trustworthy in public interest" had nothing really to do with the conclusion or arnots editorial on vexing social ills and was rather focused in a flawed manner in some other aspect of the argument, that being the gov't's ability with public interest and saw this as a distortion of the argument because it was not focused on the core of arnots case. how is the NS not distorting the argument instead of addressing the fundamental changes he is attacking the goverments ability to be functional in the first place?

am i over-complicating this?
 
raschorr
Thanks Received: 0
Forum Guests
 
Posts: 3
Joined: January 16th, 2013
 
 
 

Re: Q14 - Newspaper subscriber: Arnot's editorial argues

by raschorr Sat Jul 27, 2013 5:59 pm

i am still unclear on why A is the correct answer...

If the conclusion of the newspaper subscriber is that Arnot's conclusion is false, where is there a lack of explanation for that repudiation?

Isn't the explanation the last sentence? that "the argument Arnot makes for this claim depends on a dubious assumption..." Are those not adequate grounds?
 
ca_teran1
Thanks Received: 2
Jackie Chiles
Jackie Chiles
 
Posts: 29
Joined: May 23rd, 2012
 
 
 

Re: Q14 - Newspaper subscriber: Arnot's editorial argues

by ca_teran1 Tue Aug 06, 2013 8:17 pm

I thought "claim" meant conclusion not premise. This is why I skipped it and choose C.
C under timed pressure looks good.
 
aznriceboi17
Thanks Received: 5
Elle Woods
Elle Woods
 
Posts: 76
Joined: August 05th, 2013
 
 
trophy
Most Thankful
 

Re: Q14 - Newspaper subscriber: Arnot's editorial argues

by aznriceboi17 Wed Sep 11, 2013 12:56 am

Hi, I was thrown off by A's use of the phrase 'inadequate argument'.

For LR questions in general, my understanding was that an argument is invalid only if there is something wrong with its logical structure. We don't consider the truth values of the premises of the argument in making this evaluation (which is why in weaken the argument questions, we usually don't look at premise attackers).

If I then make a small jump from assuming 'inadequate argument' is the same as 'invalid argument', then A wouldn't be a good answer, since the subscriber has only attacked a premise of Arnot's (specifically, he's attacking the premise that government can be trusted to act in the interest of the public). Is that correct? Do 'inadequate' and 'invalid' not have the same meaning here (if that's the case, I guess 'inadequate' would be a superset of 'invalid' arguments by including valid arguments which rely on at least one premise that is false)?
 
amil91
Thanks Received: 5
Elle Woods
Elle Woods
 
Posts: 59
Joined: August 02nd, 2013
 
 
 

Re: Q14 - Newspaper subscriber: Arnot's editorial argues

by amil91 Thu Dec 05, 2013 3:11 pm

raschorr Wrote:i am still unclear on why A is the correct answer...

If the conclusion of the newspaper subscriber is that Arnot's conclusion is false, where is there a lack of explanation for that repudiation?

Isn't the explanation the last sentence? that "the argument Arnot makes for this claim depends on a dubious assumption..." Are those not adequate grounds?

I think you are mistaking the wording of the answer choice a bit. Choice A is saying he repudiates the claim on the grounds that there is insufficient evidence or an inadequate argument, for the claim he refuting, not for the claim he is making. Does that make sense?

Basically choice A is saying he says the conclusion that Arnot is making is false because his evidence is inadequate, not that the NS's argument is inadequate.

As for the person that picked C and thought this claim indicated the conclusion, this claim does indicate the conclusion. The NS says Arnot's conclusion is clearly false and goes on to say that one of Arnot's necessary assumptions is dubious, indicating it is highly unlikely to be true or practically false. Choice C does not really match up with that, for a few reasons:
Just because the NS says the conclusion is false doesn't mean it truly is, that is their argument. C would fit more for a situation where the conclusion was truly false. Additionally, we don't care if some of the assumptions may be true, that's not at stake here, and for all we know some of the other assumptions of Arnot's argument may be true.

The NS is basically saying that Arnot is wrong because his evidence is unlikely. That matches up quite well with choice A - repudiates a claim because it has an inadequate argument.
 
jones.mchandler
Thanks Received: 2
Jackie Chiles
Jackie Chiles
 
Posts: 40
Joined: February 28th, 2014
 
 
 

Re: Q14 - Newspaper subscriber: Arnot's editorial argues

by jones.mchandler Tue Aug 26, 2014 10:33 am

I just want to confirm--even though C is the incorrect answer, C does a represent a valid flaw, right? C could be a flaw that appears on the LSAT?
User avatar
 
ohthatpatrick
Thanks Received: 3808
Atticus Finch
Atticus Finch
 
Posts: 4661
Joined: April 01st, 2011
 
This post thanked 1 time.
 
 

Re: Q14 - Newspaper subscriber: Arnot's editorial argues

by ohthatpatrick Sat Aug 30, 2014 4:39 pm

Let me corral together a bunch of follow up questions:

1. Does Arnot’s argument do what the newspaper subscriber says it dubiously assumes?

answer: We have no idea! All we know is that Arnot wrote an editorial, in which he argued that changing some stuff in government would fix our social ills. We have no idea what changes Arnot wanted to make, what premises he advanced, what conclusions he drew. We only know that one of the claims Arnot argued for was that “changing certain stuff in govt. would virtually eliminate our social ills”.

2. Isn’t (D) right? Isn’t the NS distorting Arnot’s argument?

answer: see before. We have no idea! We don’t know what Arnot’s argument/editorial is. We only know one of his conclusions and one of his supposedly dubious assumptions. It’s not distorting an argument to question the validity of a premise or an assumption. Just because such an objection doesn’t tackle the conclusion head-on doesn’t mean it isn’t addressing the argument/reasoning in a legitimate way.

If Argot’s argument DID assume we can trust the argument, and that assumption IS false, then Argot’s argument/reasoning would be faulty. But that doesn’t mean that Argot’s conclusion is false.

Saying that a conclusion is valid is different from saying that the argument/reasoning used to get there is valid (see the above explanation with the sun/earth coin flip).

3. Where is the “lack of explanation”? Isn’t the last sentence an explanation?

answer: (A) doesn’t say that there was no explanation. It says there was an “inadequate argument”, which matches “the argument Argot makes for this claim rests on a dubious assumption”.

(A) is saying “just because an argument rests on a dubious assumption, you can’t reject the conclusion as false on that grounds. You can only say ‘well, you haven’t convinced me of the conclusion with THAT faulty argument’. That’s not the same position as ‘the opposite of your conclusion is true’.”

4. I thought claim meant conclusion not premise?

answer: Claim is neutral; it could mean either conclusion or premise. A claim supported by something else is a conclusion. A claim used to support something is a premise. In this case ‘claim’ referred to a conclusion Argot made that was cited in the 1st sentence.

5. Is (A) cheating by attacking the premise?

The NS attacks a dubious “assumption”, not premise. This should feel like well tread LSAT territory. You are attacking an argument if you weaken an assumption upon which it relies. Strengthen/Weaken questions frequently address the validity of an assumption. Technically, I’d say an inadequate argument isn’t much different from an invalid one. An argument is valid/adequate/sufficient if the truth of its premises logically guarantees the truth of its conclusion. An argument is invalid/inadequate otherwise. So the fact that Argot’s argument even HAD an assumption makes it logically inadequate … the fact that it’s a dubious assumption would make the argument crumble, the way negating a Necessary Assumption does.

The problem is testing the distinction between “showing that an ARGUMENT is bad” and “proving that a claim is FALSE”.

6. Is C a valid flaw?

Yes absolutely. This could be a real flaw. Here, they just reversed the wording. Here the flaw is that “even though an assumption may be false, the conclusion may be true”.
User avatar
 
Mab6q
Thanks Received: 31
Atticus Finch
Atticus Finch
 
Posts: 290
Joined: June 30th, 2013
 
This post thanked 1 time.
 
 

Re: Q14 - Newspaper subscriber: Arnot's editorial argues

by Mab6q Tue Aug 18, 2015 12:03 am

Patrick could probably take what he wrote on this thread and write a short book titled Understanding the Lack of Evidence Flaw. Well done!
"Just keep swimming"
 
erikwoodward10
Thanks Received: 9
Elle Woods
Elle Woods
 
Posts: 69
Joined: January 26th, 2014
 
 
 

Re: Q14 - Newspaper subscriber: Arnot's editorial argues

by erikwoodward10 Sat Jul 16, 2016 3:03 pm

Patrick, I actually disagree with you here. You say that:

1. Does Arnot’s argument do what the newspaper subscriber says it dubiously assumes?

answer: We have no idea!


But we actually do know that Arnot's argument does what the newspaper subscriber says. The logical nature of a flaw question tells us that we have to take the premises presented to us as valid. It is up to us to determine if the logic used in formulating a conclusion based on those premises is valid or not.

With this in mind, the newspaper subscriber says that he has identified a necessary assumption in Arnot's argument ("depends on the dubious assumption"). Given the logical nature of a flaw question, we have to accept this premise as valid--the assumption identified is both a NA and "dubious".

I think that this distinction is key in correctly understanding the logical flaw in this question--which is in the gap between identifying a NA as "dubious" and a conclusion that is "false". Given a "dubious" NA, at best we could say that the conclusion is "dubious". As identified above, the flaw in this stimulus arises in the gap between "dubious" and asserting that Arnot's conclusion is "false". We can't conclude that Arnot's argument is "false" because a NA is "dubious" (dubious means suspect, but not necessarily false). In other words, the newspaper subscriber's logic does not correctly use the negation technique for NA questions!

But unless my understanding is off, we CAN infer that Arnot's argument makes a dubious necessary assumption.

However, my question is, if the conclusion of the newspaper subscriber's argument were that "clearly his conclusion is dubious" (instead of clearly it is "false"), would the reasoning in this argument be sound?
 
KatiaK713
Thanks Received: 1
Vinny Gambini
Vinny Gambini
 
Posts: 11
Joined: April 23rd, 2017
 
 
 

Re: Q14 - Newspaper subscriber: Arnot's editorial argues

by KatiaK713 Fri Jan 19, 2018 6:40 pm

ohthatpatrick Wrote:Let me corral together a bunch of follow up questions:

1. Does Arnot’s argument do what the newspaper subscriber says it dubiously assumes?

answer: We have no idea! All we know is that Arnot wrote an editorial, in which he argued that changing some stuff in government would fix our social ills. We have no idea what changes Arnot wanted to make, what premises he advanced, what conclusions he drew. We only know that one of the claims Arnot argued for was that “changing certain stuff in govt. would virtually eliminate our social ills”.


Hi!
This is what threw me off on this question - I read the stem exactly like you described above. It doesn't sound to me like we can infer that Arnold actually relied on the "dubious assumption". The subscriber thinks Arnold did, but we don't know that for sure. The subscriber could be distorting Arnot's argument (answer D).